Question begins:
"The complex ecosystem of the North American prairie has largely been destroyed..."
I found the correct conclusion, but I got tripped up on the paraphrase. I don't see how "returning as much land as possible to an ...
Aren't there two main ways to weaken an argument? Either by going for the premises (contradicting them) or showing why the conclusion doesn't necessarily follow from them? I thought C did the first, but now I am having doubts. The stimulus concludes that ...
Someone please save me. I chose A because the author is assuming that freedom is worth more than anything else, even more than your life and I feel like A is catching onto to that by saying there could be ...
The stimulus opens with a question, (Is it correct for the gov't to abandon efforts to determine toxicity levels in food supply?) and the next sentence is the answer. How is the answer to that question the MC when the following sentence begins with " ...
I do not understand why answer choice A is an incorrect answer choice. According to the passage, more incidents of injuries occurred on the ski slopes in 1950 than occurred in 1980. The question stem then asks which answer choice most conflicts with the ...
Can someone that knows about games help me do this one in a stacked way? JY explains it in a linear way which is not how it is presented and I am confused on how to do it. Thank you.
**Admin Note: Edited title. Please use the format PT#.S#.Q ...
The hotel being at 100% capacity seems to be the right answer to me as if the hotel is at 100% capacity no amount of decorations can better this number. Upgrading the decor, and thereby price, would have an ambiguous impact ...
The question gives you an initial claim as well as a principle to go with it. The stem asks for you to pick an AC that could be appropriately used as a premise for an argument that uses the principle in the stimulus. I see this as more of a pseudo- ...
I have been going to different forums to understand this question and i'm still having trouble with it. Idk why C is wrong. Wrong timed and in BR. Thanks!
**Admin note: edited title; please use the format of "PT#.S#.Q# - [brief description ...
First of all, the conclusion. There are no conclusion indicators and I was confused between 2 statements to select for the conclusion. Secondly, answer choice (A) and (C), aren't they basically both saying the same ...
I'm having a little trouble understanding why the answer to this question is C). If the author explicitly states at the end of the passage that, "the survey data do not establish that financial problems are the major problem in contemporary marriages," how ...
Is this a correlation-causation argument because it assumes that the increase in high school dropouts is the only thing that is causing the increase in recruitment among 18 year olds? And why would the author draw such a conclusion?
I can understand how ACs A-D are incorrect. I am truly struggling to see how E is correct.
My contention here is that even if a greater proportion of crimes are reported in recent years, those independent surveys would still include all of ...
How is E correct? It appears to be supporting the conclusion not weakening it. My understanding of the argument's Conclusion is that it's telling environmentalists to relax because nature is going to adjust itself to the rising levels of the atmosphere.
I think I understand what the premises are saying, but I don't understand where the author of this stimulus even got his conclusion. If we have luggages that don't contain explosives and only one percent give false positives (alarm goes off even though ...
Why would the program care about if they have "serious problems" when they said they needed to focus on building competence just to stay on air. B sounds too vague to me
I understand why AC (A) is the correct answer because it is the best suited. However, is it really an assumption the argument depends on because if you utilise JY's negation method, you can get this:
I'm struggling to note why the E choice does not also fit the bill. Since it eliminates a potential threat to the argument - and it also would weaken the argument if it were true.